とりあえず2≦a<b≦10000の範囲について調べてみたけど
a = 459 = 3*3*3*17
b = 3125 = 5*5*5*5*5
c = 3584 = 2*2*2*2*2*2*2*2*2*7
d = 3570 = 2*3*5*7*17
のとき log(c,d) = 1.0004784530314947 が最小値になる
つまりa,bが2以上10000以下ならεが0.00047845314947より小さければε=0としたときの解と一致する